Đến nội dung

Hình ảnh

$\frac{1}{x^3y^3}+\frac{y^3}{z^3}+x^3z^3\geq \frac{1}{x^2y^2}+\frac{y^2}{z^2}+x^2z^2$

bất đẳng thức am-gm

  • Please log in to reply
Chủ đề này có 2 trả lời

#1
Jiki Watanabe

Jiki Watanabe

    Hạ sĩ

  • Thành viên
  • 63 Bài viết

Cho các số thực dương x, y, z. Chứng minh rằng $\frac{1}{x^3y^3}+\frac{y^3}{z^3}+x^3z^3\geq \frac{1}{x^2y^2}+\frac{y^2}{z^2}+x^2z^2$


Bài viết đã được chỉnh sửa nội dung bởi tienduc: 28-05-2017 - 21:00

    ~O)  Sách không đơn thuần chỉ là những trang giấy mà trong đó còn chứa đựng một thế giới mà con người luôn khao khát được khám phá ...  ^_^


#2
Baoriven

Baoriven

    Thượng úy

  • Điều hành viên OLYMPIC
  • 1423 Bài viết

Ta có: $\left\{\begin{matrix}\frac{1}{x^3y^3}+\frac{1}{x^3y^3}+1\geq \frac{3}{x^2y^2} \\ \frac{y^3}{z^3}+\frac{y^3}{z^3}+1\geq \frac{3y^2}{z^2} \\ x^3z^3+x^3z^3+1\geq 3x^2z^2 \end{matrix}\right.$.

Suy ra: $\frac{1}{x^3y^3}+\frac{y^3}{z^3}+x^3z^3\geq \frac{1}{2}(\frac{3}{x^2y^2}+\frac{3y^2}{z^2}+3x^2z^2-3)$.

Mà: $-3\geq -(\frac{1}{x^2y^2}+\frac{y^2}{z^2}+x^2z^2)$ (Theo $Cauchy$ $3$ số).

Từ đó ta có đpcm.

Dấu bằng xảy ra khi $x=y=z=1$.


Bài viết đã được chỉnh sửa nội dung bởi tienduc: 27-05-2017 - 19:57

$$\mathbf{\text{Every saint has a past, and every sinner has a future}}.$$


#3
Baodungtoan8c

Baodungtoan8c

    Binh nhất

  • Thành viên mới
  • 40 Bài viết

Cho các số thực dương x, y, z. Chứng minh rằng $\frac{1}{x^3y^3}+\frac{y^3}{z^3}+x^3z^3\geq \frac{1}{x^2y^2}+\frac{y^2}{z^2}+x^2z^2$

 

Bài này có rất nhiều cách giải , bạn có thể tha khảo thêm ở đây

 

 

https://diendantoanh...y2fracy2z2x2z2/


Học từ ngày hôm qua, sống ngày hôm nay, hi vọng cho ngày mai. Điều quan trọng nhất là không ngừng đặt câu hỏi.

                                                                                                                      Albert Einstein. 






Được gắn nhãn với một hoặc nhiều trong số những từ khóa sau: bất đẳng thức, am-gm

1 người đang xem chủ đề

0 thành viên, 1 khách, 0 thành viên ẩn danh